Calcul d'une intégrale (avec restrictions)

Fin de partie
Modifié (June 2022) dans Analyse
Je vous propose de montrer que $\displaystyle \int_0^1 \dfrac{\ln(1-x)\ln x}{1+x}dx=\dfrac{13}{8}\zeta(3)-\dfrac{1}{4}\pi^2\ln 2$

Mais il y a des restrictions.
Pas d'utilisation d'intégrales doubles, de dérivation sous le signe intégral, de développement en série. Mais l'utilisation de changement de variable et de l'intégration par parties est autorisée.
Mais vous pouvez utiliser les égalités suivantes:
\begin{align}\int_0^1 \frac{\ln x}{1-x}dx&=-\frac{1}{6}\pi^2,&\quad&&\int_0^1 \frac{\ln^2 x}{1-x}dx&=2\zeta(3)\end{align}

Réponses

  • Calli
    Modifié (June 2022)
    Bonjour,
    Voilà ce que j'ai : \begin{eqnarray*} I &:=& \int _{0} ^{1} \frac{\ln (1-x) \ln x}{1+x} \,\mathrm{d}x \\ &\overset{\text{IPP}}=& \cancel{\Big[\ln (1-x) \ln x\ln (1+x)\Big]_{0} ^{1} } -\int _{0} ^{1} \left( \frac{\ln (1-x)}{x} - \frac{\ln x}{1-x} \right) \ln (1+x) \,\mathrm{d}x\\ &=& - \underbrace{\int _{0} ^{1} \frac{\ln (1-x)\ln (1+x)}{x} \,\mathrm{d}x}_{J} + \underbrace{\int _{0} ^{1} \frac{\ln x\ln (1+x)}{1-x} \,\mathrm{d}x}_{K} \end{eqnarray*} Puis : \begin{eqnarray*} 2J &=& \int _{0} ^{1} \frac{(\ln (1-x)+\ln (1+x))^2 -\ln (1-x)^2 -\ln (1+x)^2 }{x} \,\mathrm{d}x \\ &=& \int _{0} ^{1} \frac{\ln (1-x^2 )^2 }{x} \,\mathrm{d}x - \int _{0} ^{1} \frac{\ln (1-x)^2 }{x} \,\mathrm{d}x - \int _{0} ^{1} \frac{\ln (1+x)^2 }{x} \,\mathrm{d}x\\ &\overset{u=x^2 }=& \int _{0} ^{1} \frac{\ln (1-u)^2 }{2u} \,\mathrm{d}u - \int _{0} ^{1} \frac{\ln (1-x)^2 }{x} \,\mathrm{d}x - \int _{0} ^{1} \frac{\ln (1+x)^2 }{x} \,\mathrm{d}x\\ &=& - \frac{1}{2} \int _{0} ^{1} \frac{\ln ^2 x}{1-x} \,\mathrm{d}x - \int _{0} ^{1} \frac{\ln (1+x)^2 }{x} \,\mathrm{d}x\\ &=& - \zeta (3)- \int _{0} ^{1} \frac{\ln (1+x)^2 }{x} \,\mathrm{d}x \end{eqnarray*} Et [édit : à partir de là, le calcul est faux, donc j'efface en partie] $$\cancel{  K \overset{\text{IPP}}= \dots =\ -I + \frac{\pi ^2 }{12}}  $$ Donc \[\cancel{2I = \frac{\zeta (3)}{2} + \frac{1}{2} \int _{0} ^{1} \frac{\ln (1+x)^2 }{x} \,\mathrm{d}x + \frac{\pi ^2 }{12}}.\] Ensuite il reste à calculer $\displaystyle \int _{0} ^{1} \frac{\ln (1+x)^2 }{x} \,\mathrm{d}x$ mais je ne sais pas comment faire.
    D'ailleurs, je m'étonne de ne pas avoir de $\ln2$ devant le $\pi^2$. J'espère que je n'ai pas fait d'erreur de calcul.

    PS : Bon, c'est la première fois que je me laisse tenter par ces jeux d'intégrales !
  • etanche
    Modifié (June 2022)
    Lemme 2.2 page 4 de l’article 
    Linear relations between logarithmic integrals of high weight and some-closed evaluations by Kam Cheong Au 
  • Fin de partie
    Modifié (June 2022)
    @Calli: Ton calcul est plus simple que le mien, me semble-t-il. Tu n'arrives pas non plus à éviter l'intégrale $\displaystyle \int_0^1 \dfrac{\ln x\ln(1+x)}{1+x}dx$. Je l'ai déjà traitée avec "le cahier des charges" donné sur le forum dans le passé. Si je me souviens bien il faut passer par le changement de variable $y=\dfrac{x}{1+x}$
  • @Calli: Malheureusement il y a une erreur dans ton calcul. (j'ai vérifié numériquement). un $\pi^2$ sans $\ln 2$ en facteur c'est louche en effet!
  • Calli
    Modifié (June 2022)
    Mais je viens de voir des erreurs de calculs dans ce que j'ai fait @Fin de partie (un signe $-$ dans une IPP et une utilisation de la fameuse formule $\ln a\ln b=\ln(ab)$, ou devrais-je dire "fumeuse"). J'essaie de corriger ça, si c'est rattrapable...

    Edit : Oui, c'est normal du coup que ça ne marche pas numériquement.
  • Fin de partie
    Modifié (June 2022)
    $\displaystyle \int_0^1 \dfrac{\ln^2(1+x)}{x}dx=\dfrac{1}{4}\zeta(3)$
  • Calli
    Modifié (June 2022)
    En fait, quand je corrige mon calcul de $K$, j'ai $K = I+J$. Sachant que $I=-J+K$, j'ai $0=0$ ; appelez-moi Toto.
    Donc, ce que j'ai montré normalement c'est juste que $$I = \overbrace{ \frac{\zeta (3)}{2} + \frac{1}{2} \int _{0} ^{1} \frac{\ln (1+x)^2 }{x} \,\mathrm{d}x}^{-J} +\overbrace{\int _{0} ^{1} \frac{\ln x\ln (1+x)}{1-x}  \,\mathrm{d}x}^{K}.$$ Il reste donc à calculer ces deux intégrales. Je ne sais pas si on est beaucoup plus avancés.  :D
  • @etanche : le calcul de $\displaystyle \int_0^1 \frac{\ln(1+x)^2}x\,\mathrm{d}x$ que tu as joint utilise un développement en série à la dernière étape apparemment ("calculated using geometric series"). Je ne suis pas sûr car je ne sais plus ce qu'est $\mathrm{Li}_k$. Mais si c'est le cas, alors c'est exclu par "les règles du jeu".
  • $\displaystyle \int_0^1 \dfrac{\ln^2(1+x)}{x}dx=\dfrac{1}{4}\zeta(3)$
    Je présume que ça se montre à partir de $\displaystyle \int_0^1 \frac{\ln^2(1- x)}{x}\,\mathrm{d}x=2\zeta(3)$, mais je ne sais pas comment.

    PS : Je me doutais bien qu'en me lançant là-dedans, il y aurait des valeurs d'intégrales à connaître et que ça me bloquerait. :mrgreen:
  • moinsun
    Modifié (June 2022)
    Je ne veux vraiment pas vous déranger alors je mets mon message en "caché" (mais rassurez vous c'est dans l'esprit de ce dont vous parlez) : 

    Votre jeu consiste en l'évaluation d'intégrales en s'autorisant ou pas certaines opérations. Est-il possible de "prédire" plus ou moins quand le jeu "aboutit" (c'est-à-dire, savoir quand le jeu d'opération autorisé est suffisant pour exprimer ("élémentairement" ?) l'intégrale) ?

    Ça me semble être une question peut-être (un peu) parallèle à celle d'un article de Kontsevich et Zagier (Conjecture 1, page 8) où il est question de déterminer le "jeu minimal" (?) d'opérations pour passer d'une représentation intégrale d'une période à une autre (si cette deuxième existe).

  • Fin de partie
    Modifié (June 2022)
    @Calli: Les deux égalités que j'ai indiquées dans le premier message sont utiles autrement je doute qu'on puisse y arriver avec les restrictions imposées.
    J'ai indiqué plus haut un début de piste pour faire le calcul de l'intégrale manquante.
    @Moinsun: les "périodes" dont tu parles sont, en particulier, des intégrales qui peuvent être des intégrales multiples. Ici, je veux justement ne pas avoir à utiliser d'intégrales multiples, mais, évidemment, cela se paie par la connaissance des égalités indiquées dans mon premier message.
    PS: @Calli: je pense voir une erreur dans une de tes intégration par parties.
    PS2: C'est vrai que l'espace généré par un saut de ligne est monstrueux. >:)
  • \begin{align}K&=\int_0^1 \frac{\ln(1+x)\ln x}{1-x}dx\\&\overset{\text{IPP}}=-\Big[\ln(1-x)\ln(1+x)\ln x\Big]_0^1+\int_0^1 \ln(1-x)\left(\frac{\ln(1+x)}{x}+\frac{\ln x}{1+x}\right)dx\\ &=\int_0^1 \frac{\ln(1-x)\ln(1+x)}{x}dx+\int_0^1 \frac{\ln(1-x)\ln x}{1+x}dx \end{align}

    A priori , on tourne en rond car on récupère l'intégrale qu'on veut calculer initialement mais il n'y a pas de signe moins qui la précède.
  • Néanmoins, l'intégrale $\displaystyle \int_0^1 \dfrac{\ln(1-x)\ln(1+x)}{x}dx$ est peut-être une clef pour calculer $\displaystyle \int_0^1 \dfrac{\ln(1+x)^2}{x}dx$ :)
  • Fin de partie
    Modifié (June 2022)
    Puisque $(a+b)^2-(a-b)^2=a^2+b^2+2ab-a^2-b^2+2ab=4ab$, donc
    \begin{align}A=&\int_0^1  \frac{\ln(1-x)\ln(1+x)}{x}dx\\&=\frac{1}{4}\underbrace{\int_0^1 \frac{\ln(1-x^2)^2}{x}dx}_{u=1-x^2}-\frac{1}{4}\underbrace{\int_0^1 \frac{\ln^2\left(\frac{1-x}{1+x}\right)}{x}dx}_{u=\frac{1-x}{1+x}}\\ &=\frac{1}{8}\int_0^1 \frac{\ln^2 u}{1-u}du-\frac{1}{2}\int_0^1 \frac{\ln^2 u}{1-u^2}du\\ &=\frac{1}{8}\int_0^1 \frac{\ln^2 u}{1-u}du-\frac{1}{2}\bigg(\int_0^1 \frac{\ln^2 u}{1-u}du-\underbrace{\int_0^1 \frac{u\ln^2 u}{1-u^2}du}_{z=u^2}\bigg)\\ &=-\frac{5}{16}\int_0^1 \frac{\ln^2 u}{1-u}du \end{align}
    D'un autre côté, $\displaystyle ab=\frac{1}{2}\left((a+b)^2-a^2-b^2\right)$ donc; \begin{align}A&=\frac{1}{2}\bigg(\underbrace{\int_0^1 \frac{\ln(1-x^2)^2}{x}dx}_{u=1-x^2}-\underbrace{\int_0^1 \frac{\ln^2(1-x)}{x}dx}_{u=1-x}-\int_0^1 \frac{\ln^2(1+x)}{x}dx\bigg)\\ &=\frac{1}{4}\int_0^1 \frac{\ln^2 u}{1-u}du-\frac{1}{2}\int_0^1 \frac{\ln^2 u}{1-u}du-\frac{1}{2}\int_0^1 \frac{\ln^2(1+x)}{x}dx\\ \end{align} Donc: $\boxed{\displaystyle \int_0^1 \frac{\ln^2 (1+u)}{u}du=\frac{1}{8}\int_0^1 \frac{\ln^2 u}{1-u}du=\frac{1}{4}\zeta(3)}$.
  • Calli
    Modifié (June 2022)
    @Fin de partie : Oui j'avais vu mes erreurs de calcul que tu pointes. Je les avais déjà signalées.

    Malin le coup du $(a+b)^2-(a-b)^2=4ab$ et le changement de variable $u=\frac{1-x}{1+x}$ ; je n'y avais pas pensé.
  • gai requin
    Modifié (June 2022)
    Ben alors @Calli, la méthode de Gauss pour les formes quadratiques ! ;)
    $(a,b)\mapsto 4ab$ est de signature $(1,1)$.
  • @gai requin : Franchement, j'ai enterré cet algorithme le jour où je l'ai découvert en cours de L3 (dans une fosse commune avec les tables de caractères... 💀). Pour moi, $4ab = (a+b)^2-(a-b)^2$ est juste une identité algébrique classique et je n'y avais pas pensé. J'avais $2ab = (a+b)^2-a^2-b^2$ et je ne suis pas allé assez loin dans mon idée.
  • @Calli : Sacrilège !  :D
    Pour te réconcilier avec les fq, j'ai une petite khôlle pour toi avec une question pas trop dure et une autre à laquelle je n'ai pas su répondre. 🤔
  • Fin de partie
    Modifié (June 2022)
    On peut utiliser un autre "noyau" pour calculer $\displaystyle \int_0^1 \dfrac{\ln^2\left(1+x\right)}{x}dx$
    \begin{align}B&=\int_0^1 \frac{\ln(1+x)\ln\left(\frac{x}{1+x}\right)}{x}dx\\ &=\underbrace{\int_0^1 \frac{\ln(1+x)\ln x}{x}dx}_{\text{IPP}}-\int_0^1\frac{\ln^2(1+x)}{x}dx\\ &=-\frac{1}{2}\int_0^1 \frac{\ln^2 x}{1+x}dx-\int_0^1\frac{\ln^2(1+x)}{x}dx\\ \end{align} On peut obtenir une autre expression de $B$, \begin{align} B&\overset{u=\frac{x}{1+x}}=-\int_0^{\frac{1}{2}}\frac{\ln(1-u)\ln u}{u(1-u)}du\\ &=-\int_0^{\frac{1}{2}}\frac{\ln(1-u)\ln u}{u}du-\int_0^{\frac{1}{2}}\frac{\ln(1-u)\ln u}{1-u}du\\ &\overset{\text{IPP}}=-\frac{1}{2}\Big[\ln(1-u)\ln^2 u\Big]_0^{\frac{1}{2}}- \frac{1}{2}\int_0^{\frac{1}{2}}\frac{\ln^2 u}{1-u}du+\frac{1}{2}\Big[\ln^2(1-u)\ln u\Big]_0^{\frac{1}{2}}- \frac{1}{2}\underbrace{\int_0^{\frac{1}{2}}\frac{\ln^2 (1-u)}{u}du}_{z=1-u}\\ &=- \frac{1}{2}\int_0^{\frac{1}{2}}\frac{\ln^2 u}{1-u}du- \frac{1}{2}\int_{\frac{1}{2}}^1\frac{\ln^2 z}{1-z}dz\\ &=- \frac{1}{2}\int_0^{\frac{1}{2}}\frac{\ln^2 u}{1-u}du- \frac{1}{2}\left(\int_0^1\frac{\ln^2 z}{1-z}dz-\int_0^{\frac{1}{2}}\frac{\ln^2 z}{1-z}dz\right)\\ &=- \frac{1}{2}\int_0^1\frac{\ln^2 z}{1-z}dz \end{align} Donc, \begin{align}\int_0^1\frac{\ln^2(1+x)}{x}dx&=\frac{1}{2}\left(\int_0^1 \frac{\ln^2}{1-z}dz-\int_0^1 \frac{\ln^2 x}{1+x}dx\right)\\ &=\int_0^1 \frac{x\ln^2 x}{1-x^2}dx\\ &\overset{u=x^2}=\frac{1}{8}\int_0^1 \frac{\ln^2 u}{1-z}dx=\frac{1}{8}\times 2\zeta(3)=\boxed{\frac{1}{4}\zeta(3)} \end{align}
    PS.
    Je vous laisse deviner à quoi peut servir $\displaystyle C=\int_0^1 \dfrac{\ln(1-x)\ln\left(\dfrac{x}{1-x}\right)}{x}dx$.
    A noter, la propriété curieuse: $B=C$.
  • Fin de partie
    Modifié (June 2022)
    Le challenge défi suivant est, avec les mêmes restrictions, de montrer que:
    $\displaystyle \int_0^1 \dfrac{\ln(2+x)\ln x}{1+x}dx=-\dfrac{13}{24}\zeta(3)$
    Mais pour le moment je ne sais pas faire. Peut-être qu'on ne peut pas.
    PS.
    J'ai une solution pour l'intégrale qui ouvre ce fil mais je laisse chercher ceux que cela pourrait intéresser encore un jour ou deux.
  • Calli
    Modifié (June 2022)
    Oui, laisse du temps stp @Fin de partie (surtout qu'on a une vie à côté, donc 1 ou 2 jour ça n'est pas beaucoup). J'essaierai de continuer à chercher demain. Mais je ne suis pas du tout assuré de trouver quelque chose ! Si je sèche je le dirai.

    @gai requin : Si tu veux ^^. Tu peux me l'envoyer par message privé et j'ouvrirai un fil où j'essaierai de résoudre la question pas trop dure (pas tout de suite car j'ai d'abord des intégrales à torturer ;) mais ce weekend).
  • @Calli: Je donnerai une solution la semaine prochaine.
  • Calli
    Modifié (June 2022)
    Merci @Fin de partie. Mais c'est bon j'ai trouvé ! 🥳 🎉🎊
     J'ai dû, pour m'y retrouver, passer par le calcul systématique de toutes les intégrales du type $\displaystyle \int_0 ^1 \frac{\ln(\square)\ln(\square)}\square \,\mathrm{d}x$ avec des $x$, $1-x$ et $1+x$ à la place des $\square$ (il y en a 14)... et évidemment c'est celle du fil que j'ai attrapée en dernier :mrgreen:. Mais j'ai fini par l'avoir !
    Et merci pour les exemples de calculs que tu as donnés ; j'y ai puisé pas mal d'idées.  ;)
    Je poste ma solution en soirée.
  • Calli
    Modifié (June 2022)
    Solution du calcul de $\displaystyle \int _{0} ^{1} \frac{\ln x\ln (1-x)}{1+x} \,\mathrm{d}x$ : 
    Grâce à ce message de Fin de partie, on sait que \[\textcolor{red}{\int _{0} ^{1} \frac{\ln (1-x)\ln (1+x)}{x} \,\mathrm{d}x} = - \frac{5}{8} \zeta (3) \quad\quad \text{ et } \quad\quad \textcolor{blue}{\int _{0} ^{1} \frac{\ln (1+x)^2 }{x} \,\mathrm{d}x} = \frac{\zeta (3)}{4} .\] Je le réutiliserai sans le redémontrer. Et je rappelle qu'on suppose $\textcolor{rgb(200,100,0)}{\displaystyle \int _{0} ^{1} \frac{\ln x}{1-x} \,\mathrm{d}x} =\displaystyle - \frac{\pi ^2 }{6} $. C'est parti.\begin{eqnarray*} I &:=& \int _{0} ^{1} \frac{\ln (1-x) \ln x}{1+x} \,\mathrm{d}x \\ &\overset{\text{IPP}}=& \cancel{\Big[\ln (1-x) \ln x\ln (1+x)\Big]_{0} ^{1} } -\int _{0} ^{1} \left( \frac{\ln (1-x)}{x} - \frac{\ln x}{1-x} \right) \ln (1+x) \,\mathrm{d}x\\ &=& - \textcolor{red}{\int _{0} ^{1} \frac{\ln (1-x)\ln (1+x)}{x} \,\mathrm{d}x} + \int _{0} ^{1} \frac{\ln x\ln (1+x)}{1-x} \,\mathrm{d}x \\ &=& \frac{5}{8} \zeta (3) + \ln 2 \textcolor{rgb(200,100,0)}{\displaystyle \int _{0} ^{1} \frac{\ln x}{1-x} \,\mathrm{d}x} - \textcolor{rgb(0,155,155)}{\int _{0} ^{1} \frac{\ln x\ln ( \frac{2}{1+x} )}{1-x} \,\mathrm{d}x} \end{eqnarray*} Je continue à part avec la dernière intégrale : \begin{eqnarray*} \textcolor{rgb(0,155,155)}{\int _{0} ^{1} \frac{\ln x\ln ( \frac{2}{1+x} )}{1-x} \,\mathrm{d}x} &\underset{u:= \frac{1-x}{1+x} }=& \int _{0} ^{1} \frac{\ln ( \frac{1-u}{1+u} )\ln (1+u)}{1- \frac{1-u}{1+u} } \, \frac{2\,\mathrm{d}u}{(1+u)^2 } \\ &=& \int _{0} ^{1} \frac{\ln ( \frac{1-u}{1+u} )\ln (1+u)}{u(1+u)} \, \mathrm{d}u \\ &=& \int _{0} ^{1} \frac{\ln ( \frac{1-u}{1+u} )\ln (1+u)}{u} \, \mathrm{d}u - \underbrace{\int _{0} ^{1} \frac{\ln ( \frac{1-u}{1+u} )\ln (1+u)}{1+u} \, \mathrm{d}u}_{t := \frac{1-u}{1+u} } \\ &=& \textcolor{red}{\int _{0} ^{1} \frac{\ln (1-u)\ln (1+u)}{u} \,\mathrm{d}u} - \textcolor{blue}{\int _{0} ^{1} \frac{\ln (1+u)^2 }{u} \,\mathrm{d}u} - \int _{0} ^{1} \frac{\ln t\ln ( \frac{2}{1+t} )}{1+t} \, \mathrm{d}t \\[1mm] && \qquad \textcolor{gray}{\text{ car } (1+u)(1+t)=2 \Rightarrow (1+u)\mathrm{d}t+(1+t)\mathrm{d}u=0 \Rightarrow \frac{\mathrm{d}u}{1+u} = - \frac{\mathrm{d}t}{1+t} } \\[2mm] &=& - \frac{5}{8} \zeta (3) - \frac{\zeta (3)}{4} - \underbrace{\int _{0} ^{1} \frac{\ln t\ln 2}{1+t} \, \mathrm{d}t}_{\text{IPP}} + \underbrace{\int _{0} ^{1} \frac{\ln t\ln (1+t)}{1+t} \, \mathrm{d}t}_{\text{IPP}} \\ &=& - \frac{7}{8} \zeta (3) + \ln 2 \int _{0} ^{1} \frac{\ln (1+t)}{t} \, \mathrm{d}t - \frac{1}{2} \textcolor{blue}{\int _{0} ^{1} \frac{\ln (1+t)^2 }{t} \,\mathrm{d}t}\\ &=& - \frac{7}{8} \zeta (3) + \ln 2 \bigg( \underbrace{\int _{0} ^{1} \frac{\ln (1-t^2 )}{t} \, \mathrm{d}t}_{v:=t^2 } - \int _{0} ^{1} \frac{\ln (1-t)}{t} \, \mathrm{d}t\bigg) -\frac{\zeta (3)}{8} \\ &=& - \zeta (3) + \ln 2 \left( \frac{1}{2} \textcolor{rgb(200,100,0)}{\int _{0} ^{1} \frac{\ln (1-v)}{v} \, \mathrm{d}t} - \textcolor{rgb(200,100,0)}{\int _{0} ^{1} \frac{\ln (1-t)}{t} \, \mathrm{d}t}\right) \\ &=& - \zeta (3) + \frac{\pi ^2 \ln 2}{12} \end{eqnarray*} D'où $\displaystyle I = \frac{5}{8} \zeta (3) - \frac{\textcolor{rgb(200,100,0)}{\pi ^2 }\ln 2}{\textcolor{rgb(200,100,0)}{6}} + \textcolor{rgb(0,155,155)}{\zeta (3) - \frac{\pi ^2 \ln 2}{12} } = \frac{13}{8} \zeta (3) - \frac{\pi ^2 \ln 2}4$.
  • Dans le même genre, et avec les mêmes outils, je propose de montrer que $$\displaystyle \int_0 ^1 \frac{\ln(1-x)^2}{1+x} \,\mathrm{d}x = \frac47\zeta(3)-\frac{\pi^2\ln 2}6+\frac{(\ln 2)^3}3$$ >:) (j'ai la solution cette fois-ci du coup, car elle fait partie des nombreuses intégrales que j'ai calculées pour essayer de cerner l'intégrale du fil). Je ne sais pas si tu l'as déjà celle-là @Fin de partie.
  • Fin de partie
    Modifié (June 2022)
    Ton calcul a l'air bien
    Par le changement de variable $u=\dfrac{1-x}{1+x}$ on se ramène au calcul de $\displaystyle \int_0^1 \dfrac{\ln(1+x)\ln x}{1+x}dx$ dont j'ai esquissé une méthode de calcul (avec les restrictions).
    Si tu supposes connue : $\displaystyle \quad\quad \textcolor{blue}{\int _{0} ^{1} \frac{\ln (1+x)^2 }{x} \,\mathrm{d}x} = \frac{\zeta (3)}{4}$ C'est immédiat.
    PS.
    Un détail: C'est $ \displaystyle \int_0^1\dfrac{\ln x}{1-x}dx=\dfrac{\pi^2}{6}$ qu'on suppose connue ce qui fait qu'il y a une intégration par parties en trop dans ton calcul à mon avis.
  • La preuve de $\displaystyle \int_0^1 \dfrac{\ln(2+x)\ln x}{1+x}dx=-\dfrac{13}{24}\zeta(3)$ avec les restrictions imposées est d'une toute autre difficulté semble-t-il (peut-être que c'est impossible avec ces restrictions)
  • Fin de partie a dit : 
    Un détail: C'est $ \displaystyle \int_0^1\dfrac{\ln x}{1-x}dx=\dfrac{\pi^2}{6}$ qu'on suppose connue ce qui fait qu'il y a une intégration par parties en trop dans ton calcul à mon avis.
    C'est $ \displaystyle \int_0^1\dfrac{\ln x}{1-x}\,\mathrm{d}x=-\dfrac{\pi^2}{6}$ avec un signe $-$, et je ne vois pas où il y aurait une IPP en trop.
  • Fin de partie
    Modifié (June 2022)
    La première L'IPP indiquée dans cette ligne-là: $\displaystyle \frac{5}{8} \zeta (3) - \frac{\zeta (3)}{4} - \underbrace{\int _{0} ^{1} \frac{\ln t\ln 2}{1+t} \, \mathrm{d}t}_{\text{IPP}} + \underbrace{\int _{0} ^{1} \frac{\ln t\ln (1+t)}{1+t} \, \mathrm{d}t}_{\text{IPP}} $ ne sert à rien pour moi. Le $1+x$ en dénominateur n'a pas besoin d'être remonté.
  • Fin de partie
    Modifié (June 2022)
    Pour le problème évoqué plus haut, cela se résume me semble-t-il, à démontrer, avec les restrictions indiquées, l'égalité suivante:
    $\displaystyle 6\int_0^{\frac{1}{3}}\dfrac{\ln^2 x}{1-x}dx+3\int_0^{\frac{1}{3}}\dfrac{\ln^2 x}{1+x}dx-\ln^3 3=13\zeta(3)$
    On pourrait la traduire avec des polylogarithmes.
    PS.
    Pour le moment je ne sais pas faire avec les restrictions imposées. Peut-être que c'est impossible de la sorte.
  • Ah d'accord, tu veux dire que $$\int_0^1 \frac{\ln t}{1+t} \,\mathrm{d}t = 2\underbrace{\int_0^1 \frac{\ln t}{1-t^2} \,\mathrm{d}t}_{v:=t^2} - \int_0^1 \frac{\ln t}{1-t} \,\mathrm{d}t = \left(\frac12-1\right)\int_0^1 \frac{\ln t}{1-t} \,\mathrm{d}t =\frac{\pi^2}{12}\quad ?$$ Oui, c'est plus court.
  • Oui, c'était ce que je voulais dire. Je cherche toujours à démontrer la formule que j'ai indiquée plus haut mais je n'y suis pas encore parvenu.
  • Je n'ai pas cherché cette intégrale. Je suis déjà bien content d'avoir trouvé la première. Si tu ne sais pas faire la deuxième, ça n'est pas le débutant que je suis dans ce domaine qui va y arriver.
  • @Calli: Soit c'est impossible, soit c'est une combinaison de changements de variable du type $y=\dfrac{x}{1-x},y=\dfrac{1-x}{1+x},y=\dfrac{2x}{1+x},y=\dfrac{1}{1+x}$
  • Ma solution pour le calcul de l'intégrale dans le premier message. \begin{align*} J&=\int_0^1 \frac{\ln(1-x)\ln x}{1+x}dx\\ &=\frac{1}{2}\int_0^1 \frac{\ln^2 x}{1+x}dx+\frac{1}{2}\underbrace{\int_0^1 \frac{\ln^2(1-x)}{1+x}dx}_{u=1-x}-\frac{1}{2}\underbrace{\int_0^1 \frac{\ln^2(\frac{x}{1-x})}{1+x}dx}_{u=\frac{x}{1-x}}\\ &=\frac{1}{2}\int_0^1 \frac{\ln^2 x}{1+x}dx+\frac{1}{2}\int_0^1 \frac{\ln^2 u}{2-u}du-\frac{1}{2}\int_0^\infty \frac{\ln^2 u}{(1+u)(1+2u)}du\\ &=\frac{1}{2}\int_0^1 \frac{\ln^2 x}{1+x}dx+\frac{1}{2}\int_0^1 \frac{\ln^2 u}{2-u}du-\frac{1}{2}\int_0^\infty\left( \frac{2\ln^2 u}{1+2u}- \frac{\ln^2 u}{1+u}\right)du\\ &=\frac{1}{2}\int_0^1 \frac{\ln^2 x}{1+x}dx+\frac{1}{2}\int_0^1 \frac{\ln^2 u}{2-u}du-\frac{1}{2}\int_0^1 \left( \frac{2\ln^2 u}{1+2u}- \frac{\ln^2 u}{1+u}\right)du-\frac{1}{2}\underbrace{\int_1^\infty \left( \frac{2\ln^2 u}{1+2u}- \frac{\ln^2 u}{1+u}\right)du}_{z=\frac{1}{u}}\\ &=\frac{1}{2}\int_0^1 \frac{\ln^2 x}{1+x}dx+\frac{1}{2}\int_0^1 \frac{\ln^2 u}{2-u}du-\frac{1}{2}\int_0^1 \left( \frac{2\ln^2 u}{1+2u}- \frac{\ln^2 u}{1+u}\right)du-\frac{1}{2}\int_0^1 \left( \frac{2\ln^2 z}{(2+z)z}- \frac{\ln^2 z}{(1+z)z}\right)dz\\ &=\frac{1}{2}\int_0^1 \frac{\ln^2 x}{1+x}dx+\frac{1}{2}\underbrace{\int_0^1 \frac{\ln^2 u}{2-u}du}_{z=\frac{u}{2}}-\int_0^1 \frac{\ln^2 u}{1+2u}du+\frac{1}{2}\int_0^1\frac{\ln^2 u}{2+u}du\\ &=\frac{1}{2}\int_0^1 \frac{\ln^2 x}{1+x}dx+\frac{1}{2}\int_0^{\frac{1}{2}}\frac{\ln^2(2z)}{1-z}dz-\int_0^1 \frac{\ln^2 u}{1+2u}du+\frac{1}{2}\int_0^1\frac{\ln^2 u}{2+u}du\\ &=\frac{1}{2}\int_0^1 \frac{\ln^2 x}{1+x}dx+\frac{1}{2}\left(\ln^2 2\int_0^{\frac{1}{2}}\frac{1}{1-z}dz+2\ln 2\int_0^{\frac{1}{2}}\frac{\ln z}{1-z}dz+\int_0^{\frac{1}{2}}\frac{\ln^2 z}{1-z}dz \right)-\int_0^1 \frac{\ln^2 u}{1+2u}du+\\&\frac{1}{2}\int_0^1\frac{\ln^2 u}{2+u}du\\ &=\frac{1}{2}\int_0^1 \frac{\ln^2 x}{1+x}dx+\frac{1}{2}\ln^3 2+\ln 2\underbrace{\int_0^{\frac{1}{2}}\frac{\ln z}{1-z}dz}_{=\text{A}}+\frac{1}{2}\underbrace{\int_0^{\frac{1}{2}}\frac{\ln^2 z}{1-z}dz}_{=\text{B}}-\underbrace{\int_0^1 \frac{\ln^2 u}{1+2u}du}_{=\text{C}}+\frac{1}{2}\underbrace{\int_0^1\frac{\ln^2 u}{2+u}du}_{=\text{D}}\\ A&\overset{w=\frac{z}{1-z}}=\int_0^1\frac{\ln\left(\frac{w}{1+w}\right)}{1+w}dw=\int_0^1\frac{\ln w}{1+w}dw-\frac{1}{2}\ln^2 2\\ B&\overset{w=\frac{z}{1-z}}=\int_0^1\frac{\ln^2\left(\frac{w}{1+w}\right)}{1+w}dw=\int_0^1\frac{\ln^2 w}{1+w}dw+\frac{1}{3}\ln^3 2-2\int_0^1\frac{\ln\left(1+w\right)\ln w}{1+w}dw\\ C&\overset{w=2u}=\frac{1}{2}\int_0^2 \frac{\ln^2\left(\frac{w}{2}\right)}{1+w}dw=\frac{1}{2}\int_0^1 \frac{\ln^2\left(\frac{w}{2}\right)}{1+w}dw+\frac{1}{2}\underbrace{\int_1^2 \frac{\ln^2\left(\frac{w}{2}\right)}{1+w}dw}_{z=\frac{1}{w}}\\ &=\frac{1}{2}\int_0^1 \frac{\ln^2 w}{1+w}dw+\frac{1}{2}\ln^3 2-\ln 2\int_0^1 \frac{\ln w}{1+w}dw+\frac{1}{2}\int_{\frac{1}{2}}^1 \frac{\ln^2(2z)}{(1+z)z}dz\\ &=\frac{1}{2}\int_0^1 \frac{\ln^2 w}{1+w}dw+\frac{1}{2}\ln^3 2-\ln 2\int_0^1 \frac{\ln w}{1+w}dw+\frac{1}{2}\int_{\frac{1}{2}}^1 \frac{\ln^2(2z)}{z}dz-\frac{1}{2}\left(\int_0^1 \frac{\ln^2(2z)}{1+z}dz-\int_0^{\frac{1}{2}} \underbrace{\frac{\ln^2(2z)}{1+z}dz}_{t=2z}\right)\\ &=\frac{1}{2}\int_0^1 \frac{\ln^2 w}{1+w}dw+\frac{2}{3}\ln^3 2-\ln 2\int_0^1 \frac{\ln w}{1+w}dw-\frac{1}{2}\left(\int_0^1 \frac{\ln^2 x}{1+z}dz+\int_0^1 \frac{2\ln 2\ln z}{1+z}dz+\ln^3 2-D\right)\\ &=\frac{1}{6}\ln^3 2-2\ln 2\int_0^1 \frac{\ln w}{1+w}dw+\frac{1}{2}D\\ \end{align*} Donc, \begin{align*} J&=\int_0^1 \frac{\ln^2 x}{1+x}dx+3\ln 2\int_0^1 \frac{\ln x}{1+x}dx-\int_0^1 \frac{\ln(1+x)\ln x}{1+x}dx\\ \end{align*} Les intégrales qui restent peuvent être calculées avec les restrictions imposées.
  • On peut montrer que $\displaystyle \int_0^{\frac{1}{2}} \dfrac{\ln^2 x}{1-x}dx=\dfrac{7}{4}\zeta(3)+\dfrac{1}{3}\ln^3 2$ avec les restrictions imposées.
  • Ah oui, ta solution est très différente de la mienne. Moi j'avais absolument évité de changer l'intervalle d'intégration $[0,1]$ et d'avoir des facteurs (ou arguments de logarithme) autres que $x$, $1-x$ et $1+x$. Je me suis concentré sur chercher les relations entre les intégrales sur $[0,1]$ d'un produit de logarithmes et d'inverses de $x$, $1-x$ et $1+x$. Sinon, en sortant de ce cadre, j'étais perdu (trop de possibilités ; je ne savais plus quoi faire). Donc j'ai encore rajouté des contraintes, mais ça m'a aidé.
    Mais j'ai l'impression que ma solution était plus simple (nettement moins de volume d'écritures et moins de jonglages entre différents intervalles d'intégration). :blush:
  • C'est beau... :mrgreen:


  • Fin de partie
    Modifié (June 2022)
    C'est un peu chargé comme décoration pour un mug.
    Je pense que j'ai résolu la question que j'avais laissée en suspens. S'il n'y a pas d'erreur dans mes calculs, je peux montrer avec les restrictions imposées que $\displaystyle\int_0^1\dfrac{\ln(2+x)\ln x}{1+x}dx=-\dfrac{13}{24}\zeta(3)$.
    La méthode est "simple" on casse l'intégrale en somme d'intégrales et on essaie d'exprimer celles-ci en fonction de $\displaystyle \int_0^{\frac{1}{3}}\dfrac{\ln^2 x}{1-x}dx,\int_0^{\frac{2}{3}}\dfrac{\ln^2 x}{1-x}dx,\zeta(3) $ et des constantes.
    À la fin, les intégrales $\displaystyle \int_0^{\frac{1}{3}}\dfrac{\ln^2 x}{1-x}dx,\int_0^{\frac{2}{3}}\dfrac{\ln^2 x}{1-x}dx$ disparaissent.
    Vous allez m'objecter qu'il pourrait y avoir une relation entre ces nombres qui n'est pas plus facile à démontrer que ce qu'on cherche à démontrer. Empiriquement, il n'y en a pas, c'est-à-dire que si on manipule que certains nombres il n'y a aucune chance de se retrouver avec une telle relation.
    ? \p 100
    A=intnum(x=0,1/3,log(x)^2/(1-x));
    B=intnum(x=0,2/3,log(x)^2/(1-x));
    lindep([A,B,zeta(3),log(2)^3,log(3)^3,log(2)^2*log(3),log(3)^2*log(2),log(2)*zeta(2)]) %3 = [3795679945, 8003320400, -5757361225, -6493698666, -3011341091, -28134444500, 980245917, 771051246]~
  • Fin de partie
    Modifié (June 2022)
    Le diable est dans les détails. Je croyais m'être affranchi de l'intégrale $\displaystyle \int_0^{\frac{1}{3}} \dfrac{\ln^2 x}{1+x}dx$ (*) j'avais tort.
    Je suis sur une nouvelle piste qui consiste à démontrer la relation mentionnée ci-dessus en partant de l'intégrale $\displaystyle \int_0^{\frac{2}{3}} \dfrac{\ln^2 x}{1-x}dx$(**). Je pourrais toutefois me retrouver à démontrer que $\displaystyle \int_0^{\frac{2}{3}} \dfrac{\ln^2 x}{1-x}dx=\int_0^{\frac{2}{3}} \dfrac{\ln^2 x}{1-x}dx$ et cette nouvelle tentative échouerait.

    *: Cette intégrale est omniprésente.
    **: $\dfrac{2}{3}=2\times \dfrac{1}{3}$, le premier changement de variable est $u=\dfrac{x}{2}$.
  • Après des dizaines d'heures de calcul je suis arrêté par une intégrale que je ne sais pas exprimer en fonction de $\displaystyle \int_0^{\frac{1}{3}} \dfrac{\ln^2 x}{1-x}dx,\int_0^{\frac{2}{3}} \dfrac{\ln^2 x}{1-x}dx, \int_0^{\frac{1}{3}} \dfrac{\ln^2 x}{1+x}dx,\zeta(3)$  et d'autres constantes à base de logarithmes. En même temps, si je parviens à contourner cette difficulté il y a de grandes chances que je parvienne à une égalité du type $0=0$ tandis qu'à l'heure actuelle je peux exhiber une égalité* qui n'est sans doute pas triviale.

    l'intégrale en question est $\displaystyle \int_0^{\frac{1}{2}} \dfrac{\ln(1-x)\ln x}{1+x}dx$. C'est la cousine germaine de l'intégrale qui est dans le premier message.

    *: Il y a des dilogarithmes qui interviennent dans mon calcul, je n'ai pas essayé de leur donner une bonne tête parce que j'étais préoccupé que par les intégrales trilogarithmiques  ce qui fait qu'il n'est pas immédiat de deviner une forme close pour l'intégrale ci-dessus.

  • Fin de partie
    Modifié (June 2022)
    Avec un peu de chance j'ai deviné une expression de cette intégrale:
    ? \p 50
    A=intnum(x=0,1/2,log(x)*log(1-x)/(1+x));
    lindep([A,polylog(3,1/3),polylog(3,3/4),zeta(3),log(3)*log(2)^2,log(3)^2*log(2),log(2)^3,log(3)^2,dilog(-1/3)*log(3),dilog(1/3)*log(3),dilog(-1/3)*log(2),dilog(1/3)*log(2),dilog(2/3)*log(3)]) %4 = [-24, 48, 24, -37, 0, 14, -40, 0, 16, -16, 12, 48, 16]~
    ? \p 200
    A=intnum(x=0,1/2,log(x)*log(1-x)/(1+x));
    lindep([A,polylog(3,1/3),polylog(3,3/4),zeta(3),log(3)*log(2)^2,log(3)^2*log(2),log(2)^3,log(3)^2,dilog(-1/3)*log(3),dilog(1/3)*log(3),dilog(-1/3)*log(2),dilog(1/3)*log(2),dilog(2/3)*log(3)]) realprecision = 202 significant digits (200 digits displayed) %5 = [-24, 48, 24, -37, 0, 14, -40, 0, 16, -16, 12, 48, 16]~
  • Fin de partie
    Modifié (June 2022)
    On peut préférer:
    ? \p 200
    A=intnum(x=0,1/2,log(x)*log(1-x)/(1+x));
    lindep([A,polylog(3,1/3),polylog(3,2/3),zeta(3),log(3)*log(2)^2,log(3)^2*log(2),log(3)^3,log(2)^3,dilog(1/3)*log(2),dilog(1/3)*log(3),dilog(-1/3)*log(2),dilog(-1/3)*log(3)]) %6 = [24, -96, -96, 141, 48, -30, -16, 8, 144, -192, -108, 96]~
  • Un calcul qui peut être utile pour ce que j'ai en vue: \begin{align}2\underbrace{\int_0^{\frac{1}{2}}\frac{\ln^2(1-x)}{x}dx}_{u=1-x}+2\underbrace{\int_0^{\frac{1}{2}}\frac{\ln^2(1+x)}{x}dx}_{u=\frac{1}{1+x}}&=\underbrace{\int_0^{\frac{1}{2}}\frac{\ln^2(1-x^2)}{x}dx}_{u=1-x^2}+\underbrace{\int_0^{\frac{1}{2}}\frac{\ln^2\left(\frac{1-x}{1+x}\right)}{x}dx}_{u=\frac{1-x}{1+x}}\\ 2\int_{\frac{1}{2}}^1 \frac{\ln^2 u}{1-u}du+2\int_{\frac{2}{3}}^1\frac{\ln^2 u}{u(1-u)}du&=\frac{1}{2}\int_{\frac{3}{4}}^1 \frac{\ln^2 u}{1-u}du+2\int_{\frac{1}{3}}^1 \frac{\ln^2 u}{1-u^2}du\\ \end{align} On en déduit que: \begin{align}\int_0^{\frac{3}{4}}\frac{\ln^2 x}{1-x}dx&=-2\int_0^{\frac{1}{3}}\frac{\ln^2 x}{1-x}dx+4\int_0^{\frac{1}{2}}\frac{\ln^2 x}{1-x}dx+4\int_0^{\frac{2}{3}}\frac{\ln^2 x}{1-x}dx-5\int_0^1\frac{\ln^2 x}{1-x}dx-2\times\\&\int_0^{\frac{1}{3}}\frac{\ln^2 x}{1+x}dx+2\int_0^1\frac{\ln^2 x}{1+x}dx+\frac{4}{3}\left(\frac{2}{3}\right)^3\end{align} Or, on peut démontrer avec les restrictions imposées que: \begin{align}\int_0^1\frac{\ln^2 x}{1+x}dx&=\frac{3}{4}\int_0^1\frac{\ln^2 x}{1-x}dx\\ \int_0^{\frac{1}{2}}\frac{\ln^2 x}{1-x}dx&=\frac{7}{8}\int_0^1 \frac{\ln^2 x}{1-x}dx+\frac{1}{3}\ln^3 2 \end{align} Donc, \begin{align}\int_0^{\frac{3}{4}}\frac{\ln^2 x}{1-x}dx&=-2\int_0^{\frac{1}{3}}\frac{\ln^2 x}{1-x}dx+4\int_0^{\frac{2}{3}}\frac{\ln^2 x}{1-x}dx-2\int_0^{\frac{1}{3}}\frac{\ln^2 x}{1+x}dx+\frac{4}{3}\ln^3 \left(\frac{2}{3}\right)+\frac{4}{3}\ln^3 2\end{align}
  • Fin de partie
    Modifié (June 2022)
    Quelqu'un a résolu mon problème https://math.stackexchange.com/a/4480454/186817
    Il a été plus culotté que moi en faisant le changement de variable $\displaystyle u=\dfrac{1}{x(2+x)}$ dans l'intégrale $\displaystyle \int_0^1 \dfrac{\ln^2(x(2+x))}{1+x}dx$




  • Fin de partie
    Modifié (June 2022)
    J'ai un peu simplifié le calcul mentionné dans mon message précédent: \begin{align*}
    J&=\int_0^1 \frac{\ln(2+x)\ln x}{1+x}dx\\
    3J&=\underbrace{\int_0^1\frac{\log^2\big(x(2+x)\big)}{1+x}dx}_{u=\frac{1}{x(2+x)}}-\frac{1}{2}\underbrace{\int_0^1\frac{\log^2\left(\frac{x}{2+x}\right)}{1+x}dx}_{u=\frac{x}{2+x}}-\frac{1}{2}\int_{0}^{1}\frac{\log^2 x}{1+x}dx-\frac{1}{2}\underbrace{\int_{0}^{1}\frac{\log^2(2+x)}{1+x}dx}_{u=\frac{1}{2+x}}\\
    &=\frac{1}{2}\int_{\frac{1}{3}}^\infty \frac{\ln^2 u}{u(1+u)}du-\int_0^{\frac{1}{3}}\frac{\ln^2 u}{1-u^2}du-\frac{1}{2}\int_{0}^{1}\frac{\log^2 x}{1+x}dx-\frac{1}{2}\int_{\frac{1}{3}}^{\frac{1}{2}}\frac{\ln^2 u}{u(1-u)}du\\
    &=\frac{1}{2}\bigg(\underbrace{\int_1^\infty \frac{\ln^2 u}{u(1+u)}du}_{z=\frac{1}{u}}+\int_{\frac{1}{3}}^{1}\frac{\ln^2 u}{u}du-\int_0^1 \frac{\ln^2 u}{1+u}du+\int_0^{\frac{1}{3}}\frac{\ln^2 u}{1+u}du\bigg)-\frac{1}{2}\int_0^{\frac{1}{3}}\frac{\ln^2 u}{1-u}du\\
    &\qquad-\frac{1}{2}\int_0^{\frac{1}{3}}\frac{\ln^2 u}{1+u}du-\frac{1}{2}\int_{0}^{1}\frac{\log^2 x}{1+x}dx-\frac{1}{2}\left(\int_{\frac{1}{3}}^{\frac{1}{2}}\frac{\ln^2 u}{u}du+\int_0^{\frac{1}{2}}\frac{\ln^2 u}{1-u}du-\int_0^{\frac{1}{3 }}\frac{\ln^2 u}{1-u}du\right)\\
    &=\frac{1}{6}\ln^3 2-\frac{1}{2}\int_0^1 \frac{\ln^2 u}{1+u}du-\frac{1}{2}\int_0^{\frac{1}{2}} \frac{\ln^2 u}{1-u}du .
    \end{align*} Les deux intégrales restantes peuvent être calculées avec les restrictions imposées.
Connectez-vous ou Inscrivez-vous pour répondre.